LSAT and Law School Admissions Forum

Get expert LSAT preparation and law school admissions advice from PowerScore Test Preparation.

 PhilD
  • Posts: 7
  • Joined: Apr 05, 2012
|
#3928
Dec 2004 practice test, LR Section 1, problem 21

This problem is a Justify problem. Two of the answer choices, A and B, are very similar conditional statements where the big difference seems to be the scope of the necessary conditions. Choice A says "...ALL major meteor impacts" and Choice B says "...MANY major meteor impacts". The correct answer choice, A, is stronger than choice B.

Will all Justify answers contain strong language like this ('all' vs. 'many') since the answer choice is sufficient for the conclusion in the stimulus?
User avatar
 Dave Killoran
PowerScore Staff
  • PowerScore Staff
  • Posts: 5852
  • Joined: Mar 25, 2011
|
#3942
Hi Phil,

Not necessarily. Although all Justify questions have to be airtight (and thus certain), where and how that certainty appears can vary. For example, does that mean that the strong language you mention has to appear in the answer choice? No. Here's an example:

  • Premise: If some people choose the chicken entree, then everyone must choose the chicken entree.

    Conclusion: Everyone must choose the chicken entree.

    Correct answer: Some people choose the chicken entree.

So, the correct answer does not contain the "strong" language we were discussing, but that's ok because the conclusion already handled it.

There are also ways to construct airtight arguments without obvious extreme language, so even strong language itself is not a necessity.

I hope that helps. Thanks!
 jiyounglee
  • Posts: 25
  • Joined: Aug 10, 2016
|
#27902
Would you be able to elaborate further why D is wrong?

causal link between major meteor impacts and mass extinctions :arrow: all mass extinctions could have followed major meteor impacts

Is it because sufficient condition is that there is no causal link therefore it may or may not happen based on necessary condition?
 Adam Tyson
PowerScore Staff
  • PowerScore Staff
  • Posts: 5153
  • Joined: Apr 14, 2011
|
#27931
I'm going to start by giving you a simple, straightforward answer. Yes!

Your analysis appears correct to me - the necessary condition in answer D could happen even if the sufficient condition did not.

Looked at another way, answer D is the opposite of what we want here. We want to justify the conclusion that there is no consistent causal link. To show that using a conditional statement, we want that conclusion to be necessary (justified, proven, must be true), and in D that claim is merely sufficient.

Good eye!
 ChicaRosa
  • Posts: 111
  • Joined: Aug 23, 2016
|
#28320
Can anyone explain why A is the correct answer choice out of all the other choices?

I was struggling with this one because the first and second premises uses the word "many" which is the same word as "some' to show that some mass extinctions didn't follow any meteor impacts and vice versa. Then the conclusion says that there are no causal link between the two events.

The two assumptions are:

1. If some extinctions didn't follow any known major meteor impacts, then there is no causal link between them.
2. If some major meteor impacts didn't follow any extinctions, then there is no causal link between them.

Yet when I did the contrapositive of my assumptions I ended up with this:

1.If there is a causal link between them then some extinctions did follow any known major meteor impacts.
2. If there is a causal link between them then some major meteor impacts did follow any extinctions.

Which led me to answer choice B because the word "many" in the answer choice.

Is my line of reasoning right?

Thanks!
User avatar
 Jonathan Evans
PowerScore Staff
  • PowerScore Staff
  • Posts: 726
  • Joined: Jun 09, 2016
|
#28460
Hi, Chica,

Good question on a difficult problem.

Remember on a Justify problem we are looking for an assumption that would be sufficient to prove the conclusion is valid. There could be a variety of necessary assumptions involved. Your job here is to add information to the premises that in combination with the other facts would justify the conclusion.

In other words, your work for a stimulus such as this might look something like this:

P: There exist (some/many) EXT not preceded by MMI.
P: There exist (some/many) MMI not followed by EXT.

C: There is no causal link between MMI and EXT.

Prephrase: We need something sufficient to prove that the fact that not all EXT and MMI occur together is itself sufficient to prove that there is no causal link between MMI and EXT.

If not all MMI and EXT together :arrow: no causal link between MMI and EXT

In contrapositive form, if there were a causal link between MMI and EXT, then all MMI and EXT would occur together. Notice the negation of "not all" to "all" in the contrapositive.

If causal link between MMI and EXT :arrow: all MMI and EXT together

This is answer choice A.

With respect to B, let's break that one down:

If causal link between MMI and EXT :arrow: there exist (some/many) MMI and EXT together

In contrapositive form, this is:

If no MMI and EXT together :arrow: no causal link between MMI and EXT

This is insufficient to justify the conclusion since there seem to be some MMI and EXT that have occurred in tandem. Note also Dave's explanation above that stipulates that quantifiers such as "many" and "all" are necessary for different situations, and "many" in premises does not imply that we need "many" in the credited response. Also be sure to note the manner in which these quantifier words act when they are negated in contrapositives (and otherwise). I know this was rather dense, but I hope it helps.
 fmbb
  • Posts: 3
  • Joined: Apr 07, 2018
|
#49965
I thought this was an assumption question not a justify question...Can someone explain what i'm missing?
 James Finch
PowerScore Staff
  • PowerScore Staff
  • Posts: 943
  • Joined: Sep 06, 2017
|
#50281
Hi FMBB,

There are two types of assumptions tested on the LSAT: sufficient and necessary. Sufficient assumptions act as sufficient conditions for the conclusion to be true: if the sufficient assumption is true, then the conclusion must also be true. These are what we call Justify questions, and generally include phrases like "can be properly inferred," "justify," or "properly drawn" in their question stems. Here we see the "properly inferred" phrase used, indicating a Justify question.

Necessary assumptions, on the other hand, act as necessary conditions to the conclusion's sufficient condition, meaning their existence enables the conclusion to be true, but doesn't necessarily make it so. They are generally recognizable by the inclusion of necessary condition indicators like "must," "relies upon," "depends upon" etc. in their question stems. The main thing to understand about them is that whenever a necessary assumption is false, the conclusion must be false as well.

You may also read more about sufficient and necessary assumptions here.

Hope this helps!
User avatar
 ToadKing
  • Posts: 18
  • Joined: Jan 17, 2021
|
#85714
Are there other justify questions similar to this one that you could point me to? This question in particular stumped me, and I still don't know whether I completely understand it. It seemed like the answer was just the contrapositive of the premises/conclusion put into a conditional format.

Thanks!
User avatar
 KelseyWoods
PowerScore Staff
  • PowerScore Staff
  • Posts: 1079
  • Joined: Jun 26, 2013
|
#85756
Hi ToadKing!

It's fairly common in Justify/Sufficient Assumption questions for the correct answer to be some variation of "If the premises are true, then the conclusion is true." Basically in a Justify question you're really just looking to draw that direct line between the premises and the conclusion. The author is trying to say that the premises prove the conclusion, but you need to help them out by finding an answer choice that says yeah, if the premises are true, then the conclusion is true. Sometimes that might feel a little redundant. But that "if premises, then conclusion" rule is what ties everything together and makes the argument rock solid. Without it, the author hasn't provided enough proof of their conclusion.

Just check out more Justify questions in general to further get the hang of these!

You may also want to check out this podcast that covers them: https://blog.powerscore.com/lsat/lsat-p ... questions/

And this blog post: https://blog.powerscore.com/lsat/unders ... questions/.

Hope this helps!

Best,
Kelsey

Get the most out of your LSAT Prep Plus subscription.

Analyze and track your performance with our Testing and Analytics Package.